LSAT and Law School Admissions Forum

Get expert LSAT preparation and law school admissions advice from PowerScore Test Preparation.

User avatar
 Dave Killoran
PowerScore Staff
  • PowerScore Staff
  • Posts: 5853
  • Joined: Mar 25, 2011
|
#59726
Complete Question Explanation
(The complete setup for this game can be found here: lsat/viewtopic.php?t=6078)

The correct answer choice is (C)

If L sits next to N, a P-N-L sequence is created:

pt1_j91_g1_q6a.png
The three remaining variables are K, M, and O. Since K and M cannot sit next to each other, by Hurdling the Uncertainty we can infer that O must separate K and M:

pt1_j91_g1_q6b.png
Since O is between K and M, answer choice (C) is correct.
You do not have the required permissions to view the files attached to this post.

Get the most out of your LSAT Prep Plus subscription.

Analyze and track your performance with our Testing and Analytics Package.